Difference between revisions of "Talk:2021 Fall AMC 10A Problems/Problem 19"

m
 
Line 5: Line 5:
  
 
Edit: Thanks MRENTHUSIASM!
 
Edit: Thanks MRENTHUSIASM!
 +
 +
Yay! :D ~MSAISUHTNERM [[File:Victorious-Hedgehog.png|30px]]

Latest revision as of 13:07, 4 December 2021

An asymptote diagram for my solution would be appreciated. You can take credit for the diagram!

Note: I cannot do asymptote diagrams. from MathFun1000

Edit: Thanks MRENTHUSIASM!

Yay! :D ~MSAISUHTNERM Victorious-Hedgehog.png